Last visit was: 11 May 2024, 06:01 It is currently 11 May 2024, 06:01

Close
GMAT Club Daily Prep
Thank you for using the timer - this advanced tool can estimate your performance and suggest more practice questions. We have subscribed you to Daily Prep Questions via email.

Customized
for You

we will pick new questions that match your level based on your Timer History

Track
Your Progress

every week, we’ll send you an estimated GMAT score based on your performance

Practice
Pays

we will pick new questions that match your level based on your Timer History
Not interested in getting valuable practice questions and articles delivered to your email? No problem, unsubscribe here.
Close
Request Expert Reply
Confirm Cancel
SORT BY:
Kudos
Math Expert
Joined: 02 Sep 2009
Posts: 93159
Own Kudos [?]: 622814 [16]
Given Kudos: 81832
Send PM
Math Expert
Joined: 02 Sep 2009
Posts: 93159
Own Kudos [?]: 622814 [4]
Given Kudos: 81832
Send PM
Intern
Intern
Joined: 10 Jan 2017
Status:Don't watch the clock,Do what it does, Keep Going.
Posts: 18
Own Kudos [?]: 10 [3]
Given Kudos: 67
Send PM
Math Expert
Joined: 02 Sep 2009
Posts: 93159
Own Kudos [?]: 622814 [2]
Given Kudos: 81832
Send PM
Re: M20-08 [#permalink]
2
Kudos
Expert Reply
Shiprasingh1100 wrote:
I am struggling a bit to understand option B. How are we approaching it?


Statement (2) by itself is insufficient. If \(y = 1\), the answer is "yes"; if \(y = 2\), the answer is "no".


If y = 1, then 10^x + 1 could be 11, 101, 1001, ... so as given in the second statement, not divisible by 9. In this case, 10^x - 1 will be 9, 99, 999, ... so divisible by 9. Answer YES.

If y = 2, then 10^x + 1 could be 12, 102, 1002, ... so as given in the second statement, not divisible by 9. In this case, 10^x - 1 will be 8, 98, 998, ... so NOT divisible by 9. Answer NO.
Manager
Manager
Joined: 21 Jul 2017
Posts: 154
Own Kudos [?]: 116 [2]
Given Kudos: 143
Location: India
Concentration: Social Entrepreneurship, Leadership
GMAT 1: 660 Q47 V34
GPA: 4
WE:Project Management (Education)
Send PM
Re: M20-08 [#permalink]
2
Kudos
Shiprasingh1100 wrote:
I am struggling a bit to understand option B. How are we approaching it?


Statement (2) by itself is insufficient. If \(y = 1\), the answer is "yes"; if \(y = 2\), the answer is "no".


The question is asking if 10^x-y is divisible by 9. Since, for x=y=1; The answer is Yes

and for x=2, y=9; The answer is No.

Hence the option is not by itself sufficient.
Math Expert
Joined: 02 Sep 2009
Posts: 93159
Own Kudos [?]: 622814 [5]
Given Kudos: 81832
Send PM
Re: M20-08 [#permalink]
1
Kudos
4
Bookmarks
Expert Reply
Galiya wrote:
Does this rule hold true for any integer?

Quote:
(something not divisible by 3) - (something divisible by 3) = (something not divisible by 3)


Yes.

WHEN THE SUM OR THE DIFFERENCE OF NUMBERS IS A MULTIPLE OF AN INTEGER

1. If integers \(a\) and \(b\) are both multiples of some integer \(k>1\) (divisible by \(k\)), then their sum and difference will also be multiples of \(k\) (divisible by \(k\)):

Example: \(a=6\) and \(b=9\), both divisible by 3 ---> \(a+b=15\) and \(a-b=-3\), again both divisible by 3.


2. If out of integers \(a\) and \(b\), one is a multiple of some integer \(k>1\) and the other is not, then their sum and difference will NOT be multiples of \(k\) (divisible by \(k\)):

Example: \(a=6\), divisible by 3, and \(b=5\), not divisible by 3 ---> \(a+b=11\) and \(a-b=1\), neither is divisible by 3.


3. If integers \(a\) and \(b\) both are NOT multiples of some integer \(k>1\) (divisible by \(k\)), then their sum and difference may or may not be multiples of \(k\) (divisible by \(k\)):

Example: \(a=5\) and \(b=4\), neither is divisible by 3 ---> \(a+b=9\), is divisible by 3, and \(a-b=1\), is not divisible by 3;
OR: \(a=6\) and \(b=3\), neither is divisible by 5 ---> \(a+b=9\) and \(a-b=3\), neither is divisible by 5;
OR: \(a=2\) and \(b=2\), neither is divisible by 4 ---> \(a+b=4\) and \(a-b=0\), both are divisible by 4.

For more on this check:

5. Divisibility/Multiples/Factors



For other subjects:
ALL YOU NEED FOR QUANT ! ! !
Ultimate GMAT Quantitative Megathread
Intern
Intern
Joined: 10 May 2017
Posts: 9
Own Kudos [?]: 13 [0]
Given Kudos: 0
Send PM
Re: M20-08 [#permalink]
Dear Sir,

I am having a doubt which might sound silly. Hence my apologies.

From the question, i understand is ((10^x)-y)/9?

Is it okay if I re-write this way: (10^x/9)-y/9 since they will have comment denominator?

Is that case, since x and y are positive, wouldn't the stem of the question itself denotes that it is not divisible by 9?
Math Expert
Joined: 02 Sep 2009
Posts: 93159
Own Kudos [?]: 622814 [0]
Given Kudos: 81832
Send PM
Re: M20-08 [#permalink]
Expert Reply
Manoraaju wrote:
Dear Sir,

I am having a doubt which might sound silly. Hence my apologies.

From the question, i understand is ((10^x)-y)/9?

Is it okay if I re-write this way: (10^x/9)-y/9 since they will have comment denominator?

Is that case, since x and y are positive, wouldn't the stem of the question itself denotes that it is not divisible by 9?


Yes, you can rewrite \(\frac{10^x-y}{9}\) as \(\frac{10^x}{9}-\frac{y}{9}\). But how can you say that \(\frac{10^x}{9}-\frac{y}{9}\) will not be an integer?

For example, if x = 1 and y = 1, then \(\frac{10^x}{9}-\frac{y}{9}=1\)
Manager
Manager
Joined: 16 Jan 2011
Posts: 72
Own Kudos [?]: 723 [0]
Given Kudos: 15
Send PM
Re: M20-08 [#permalink]
Does this rule hold true for any integer?

Quote:
(something not divisible by 3) - (something divisible by 3) = (something not divisible by 3)
Intern
Intern
Joined: 28 Mar 2017
Posts: 4
Own Kudos [?]: 0 [0]
Given Kudos: 45
Send PM
Re: M20-08 [#permalink]
I am struggling a bit to understand option B. How are we approaching it?


Statement (2) by itself is insufficient. If \(y = 1\), the answer is "yes"; if \(y = 2\), the answer is "no".
Math Expert
Joined: 02 Sep 2009
Posts: 93159
Own Kudos [?]: 622814 [0]
Given Kudos: 81832
Send PM
Re: M20-08 [#permalink]
Expert Reply
I have edited the question and the solution by adding more details to enhance its clarity. I hope it is now easier to understand.
GMAT Club Bot
Re: M20-08 [#permalink]
Moderator:
Math Expert
93159 posts

Powered by phpBB © phpBB Group | Emoji artwork provided by EmojiOne